ZF cohérent avec grands ordinaux : preuve ? — Les-mathematiques.net The most powerful custom community solution in the world

ZF cohérent avec grands ordinaux : preuve ?

Bonjour à tous,

Dans cet article d'Aaronson, à la page 6, il est écrit que ZF + LC, où LC affirme l'existence d'un grand ordinal, prouve la cohérence de ZF.

Savez-vous à qui est dû ce résultat et où je pourrais en trouver une preuve ? J'ai un peu cherché mais pas au bon endroit apparemment.

Réponses

  • Je n'y connais rien, je vais juste répéter ce que je crois être un bruit de couloir. Si tu es pressée ou pressé, cela peut peut-être t'aider ; et sinon, mieux vaut attendre la réponse des spécialistes.
    Je crois que c'est classique et facile, quand on sait de quoi on parle. Il me semble qu'un grand cardinal, quand on le regarde muni de l'appartenance, est tout simplement un modèle de $ZF$. Et donc, $LC$ est un énoncé qui postule l'existence d'un truc témoignant directement de la consistance de $ZF$.
  • Je ne suis ni pressée ni pressé, donc pas de problème.

    Merci pour cette réponse :) , je ne connaissais pas cette caractéristique.. Enfin je dois avouer que je n'ai jamais eu de cours sur des grands cardinaux, tout ce que je connais d'eux c'est quelques propriétés rapides que l'on peut trouver sur Wikipédia.

    Je vais donc peut-être attendre la réponse de spécialiste..
  • Je voudrais juste nuancer un peu le message de Georges Abitbol, certains grands cardinaux fournissent des modèles de ZF, mais ce n'est pas non plus leur définition. Tout ce que cela nous apprend est que si ZF est consistente alors ZF ne prouve pas l'existence de tels cardinaux (second théorème d'incomplétude de Gödel, et équivalence entre consistance et existence d'un modèle).
  • Je ne sais pas à qui est dû ce résultat; mais il est présent dans le tome 2 de Logique Mathématique de Cori et Lascar, ainsi que dans Théorie des Ensembles, de Jean-Louis Krivine par exemple. Enfin en supposant que par "grands ordinaux" tu veuilles dire "grands cardinaux". D'ailleurs je vais aussi faire semblant que tu as dit ZFC, parce qu'en général les hypothèses de grands cardinaux s'expriment mieux avec l'axiome du choix

    L'idée est la suivante : les grands cardinaux sont en général en particulier des cardinaux inaccessibles : un cardinal indénombrable $\kappa$ est inaccessible s'il est régulier (c'est à dire qu'il n'est pas une union indexée par un cardinal strictement plus petit d'ensembles de cardinaux strictement plus petits) et fortement limite (si $\lambda<\kappa, 2^\lambda<\kappa$). Ces hypothèses font que, intuitivement, $\kappa$ "ne peut pas être atteint" par des "opérations ensemblistes classiques" (contrairement par exemple à $\omega_1$, plus généralement $\omega_n$, ou encore $\aleph_{\omega^2}$) à partir de cardinaux plus petits.
    C'est cela qui permet de montrer que $V_\kappa$ (et non pas $\kappa$ lui-même, George Abitbol ;-) ) est un modèle de ZFC, où les $V_\alpha$ sont définis par induction par $V_\alpha = \displaystyle\bigcup_{\beta<\alpha}\mathcal{P}(V_\beta)$.

    En fait, pour n'importe quel ordinal limite $\alpha$, $V_\alpha$ est déjà "presque un modèle de ZFC" : c'est un modèle de la théorie ZC, qui est la même que ZFC sans le schéma d'axiomes de remplacement (mais avec le schéma d'axiomes de compréhension).

    Le fait que $\kappa$ soit inaccessible permet d'obtenir le remplacement, et quand on déroule la preuve, c'est assez clair pourquoi.

    Si tu veux je pourrai faire la preuve de tout ce que j'ai affirmé là
  • @Maxtimax : Aïe, aïe, aïe, il me manquait donc un morceau important... Merci pour tes précisions !
  • Bonjour.

    Comme je ne sais pas ce que tu sais sur les $V_{\alpha}$, il est peut-être utile de signaler que ces objets et leur utilité est décrite en détail dans le Dehornoy qui vient de sortir. Grosso modo, pour obtenir un modèle "encore plus beau que le précédent", il faut imaginer des ordinaux "encore plus gros que les précédents", et donc des cardinaux de plus en plus "lointains" (beau, gros et lointains étant ici des pas-concepts informels).

    Cordialement, Pierre.
  • Merci pour vos réponses !

    Oups, j'étais pourtant persuadé d'avoir écrit cardinaux... Et l'article que j'ai posté parlait bien de ZF, d'où ma non-considération pour l'axiome du choix en premier lieu.

    Martimax, si la preuve n'est pas trop longue, je veux bien une esquisse. Si c'est un peu fatiguant, pas la peine de t'embêter !

    Et maintenant que vous me parlez de Dehornoy, il me semble bien que j'ai un pdf de lui quelque part qui parle de ZFC et de grands cardinaux, je vais chercher ça.
  • La preuve n'est pas si longue. Il fait juste que je sache ce que tu sais des $V_\alpha$ ? (Si tu en sais quoi que ce soit)
  • $V_\alpha = \left\{
    \begin{array}{ll}
    \emptyset & \mbox{si } \alpha = 0\\
    \mathcal{P}(V_\beta) & \mbox{si } \alpha = \beta + 1\\
    \bigcup_{\beta\in\alpha}V_\beta & \mbox{si } \alpha \mbox{ est limite.}
    \end{array}
    \right.$, non ?
  • Je ne suis pas intervenu car je ne connais pas "LC" :-D

    Mais pour info, la consistance de ZF est "une goutte d'eau" à côté de la force ajoutée par le moindre axiome de grand cardinal. Et, il n'y a rien à prouver (enfin, ne connaissant pas "LC", va savoir...) par exemple le plus petit des "grands cardinaux", à savoir un ensemble inaccessible est "EVIDEMMENT" un modèle de ZF.

    Maintenant attention: est-ce que tu sais que quand une théorie a un modèle ALORS elle est consistante? ;-)
    Aide les autres comme toi-même car ils sont toi, ils sont vraiment toi
  • @incohérence : oui mais connais-tu leurs propriétés ?
    @christophe : c'est "évident" mais il faut quand même faire la preuve :-D
  • Hum non je dois avouer que je ne connais aucune propriété !
  • @max: si à la place "d'inaccessible" (qui veut essentiellement dire "modèle plein de ZF" ) on avait décide de prendre "modèle de ZF"? Me dirais tu "c'est évident mais?"

    Ce que je veux dire par la est que le mot "inaccessible" est une abréviation. ZF demande que l'axiome de remplacement soit vrai pour toutes les collections définissables "inac" demande qu'il le soit pour toutes tout court. De mon téléphone. Mais je respecte les difficultés de cohérence.
    Aide les autres comme toi-même car ils sont toi, ils sont vraiment toi
  • @incoherence : bon dans ce cas ça risque d'être un peu long. Je te fais une esquisse et tu me demanderas d'approfondir ce que tu n'arrives pas à prouver :
    Chaque $V_\alpha$ est transitif, et $V_\alpha \subset V_\beta$ si et seulement si $\alpha <\beta$.

    Si $x\subset V_\alpha$, alors $x\in V_{\alpha +1}$ .

    Si $\alpha$ est limite et $x\in V_\alpha$, il existe $\lambda < \alpha$ tel que $x\in V_\lambda$.

    $V_\alpha$ est un modèle de ZC, pour $\alpha$ limite $>\omega$ :
    Axiome d'extensionnalité : si $x, y\in V_\alpha$, par transitivité, $x,y \subset V_\alpha$, donc si $x\cap V_\alpha = y\cap V_\alpha$, $x=y$.
    Axiome de la paire : Si $x,y \in V_\alpha$, alors il existe $\delta <\alpha$ tel que $x,y \in V_\delta$, et donc $\{x,y\} \subset V_\delta$, donc $\{x,y\}\in V_{\delta +1}$, or $\delta +1< \alpha$ ($\alpha$ limite), donc $\{x,y\} \in V_\alpha$.
    Axiome de la réunion : Si $x\in V_\alpha$, $x\in V_\delta$ pour un $\delta < \alpha$. Par transitivité, $z\in y \in x \implies z \in V_\delta$, de sorte que $\cup x \subset V_\delta$, et donc $\cup x \in V_{\delta +1} \subset V_\alpha$, $\cup x \in V_\alpha$.
    Axiome des parties : Si $x\in V_\alpha$, $x\in V_\delta$ pour un $\delta < \alpha$. Alors $x\subset V_\delta$, donc $\mathcal{P}(x) \subset \mathcal{P}(V_\delta) = V_{\delta +1}$, et donc $\mathcal{P}(x) \in V_{\delta +2}$. Or $\alpha$ est limite donc $\delta +2 < \alpha$, et donc $\mathcal{P}(x) \in V_\alpha$.
    Axiome de l'infini : Par récurrence sur $\beta, \beta \in V_{\beta +1}$, donc $\omega \in V_{\omega +1}$. $\alpha$ est limite et $>\omega$, donc $\omega +1 <\alpha$, donc $\omega \in V_\alpha$.
    Schéma d'axiomes de compréhension : soit $\phi(y, \overline{w})$ une formule du premier ordre, et $x, \overline{w} \in V_\alpha$. Alors $x\in V_\delta$ pour un $\delta < \alpha$, de sorte que $x\subset V_\delta$, et donc $\{y\in x \mid \phi(y, \overline{w}) \} \subset V_\delta$, donc $\{y\in x \mid \phi(y, \overline{w}) \} \in V_{\delta +1} \subset V_\alpha$.
    Axiome du choix : Soit $x \in V_\alpha$ dont les éléments sont non vides et deux à deux disjoints. Alors $x\in V_\delta$ pour un $\delta <\alpha$. Soit $y$ donné par l'axiome du choix dans l'univers ambiant, tel que pour tout $z\in x$, $z\cap y$ est un singleton. Quitte à considérer son intersection avec $\cup x$, on peut supposer $y\subset \cup x$. Donc $y\subset V_\delta$, donc $y\in V_{\delta +1}$, donc $y\in V_\alpha$

    Il est assez clair à chaque fois que les témoins de vérification de l'axiome en question dans le modèle choisi initialement en sont aussi dans $V_\alpha$, pour peu qu'ils y appartiennent, donc les preuves que j'ai faites suffisent.

    Donc en fait la "seule différence" entre un inaccessible et un ordinal limite à ce niveau là, c'est le schéma d'axiomes de remplacement.

    Bon ici il y a un détail technique à prendre en compte selon le théorème que l'on veut montrer. Si on veut montrer "pour $\kappa$ inaccessible dans un modèle transitif $V$ de ZFC, $V_\kappa$ est un modèle de ZFC" (qui est essentiellement un théorème de théorie des modèles; ce qu'on appelle parfois "métamathématiques"- qui permet de montrer que si ZF est cohérente, alors ZF + "il n'existe pas d'inaccessible" l'est aussi); alors il n'y a quasiment rien à faire. Si on veut montrer le théorème de ZFC "Pour tout cardinal inaccessible, $V_\kappa$ est un modèle de ZFC" (qui est le théorème qui permet de montrer que, si ZFC est cohérente, alors ZFC ne prouve pas "Si ZFC est cohérente alors ZFC + il existe un inaccessible est cohérente"); il faut faire attention que l'on veut prouver le schéma d'axiomes de remplacement "interne", au sens où les formules que l'on doit considérer sont bien des objets de "l'univers ambiant", et pas des objets syntactiques. En fait ce n'est pas très compliqué; mais je ne m'attarderai pas sur ce genre de détails ici et je ferai la première version.

    Pour le faire proprement donc (et éventuellement voir la subtilité), je me mets en mode théorie des modèles:

    Soit $(V,\in)$ un modèle transitif de ZFC, $\kappa \in V$ tel que $V\models \kappa$ est in accessible; et $V_\kappa$ le sous-ensemble de $V$ défini par la formule usuelle. On a déjà montré que $(V_\kappa, \in) \models ZC$. Soit alors $\phi(y, z, \overline{w})$ une formule du premier ordre telle que $V_\kappa \models \forall \overline{w}, \forall y \forall z \forall z', \phi(y, z, \overline{w}) \land \phi(y, z', \overline{w}) \implies z=z'$.

    Il est alors clair que $V\models \forall \overline{w} \forall y \forall z \forall z', (\overline{w}\in V_\kappa, y \in V_\kappa, z\in V_\kappa \land \phi^{V_\kappa}(y, z, \overline{w})) \land (\overline{w}\in V_\kappa, y \in V_\kappa, z\in V_\kappa \land phi^{V_\kappa}(y, z', \overline{w})) \implies z=z'$, où $\phi^{V_\kappa}$ désigne la relativisée de $\phi$ à $V_\kappa$ (je pourrai le redéfinir si tu ne connais pas ça)

    Soit ensuite $x, \overline{w} \in V_\kappa$. On veut montrer $(V_\kappa, \in) \models \exists a, \forall t, t\in a \iff \exists y \in x, \phi(y,t,\overline{w})$

    Or $(V,\in) \models ZFC$, et par la remarque qui précède, on peut appliquer le schéma d'axiomes de remplacement à $V$ et à $\psi(y,z, \overline{w}) = (\overline{w}\in V_\kappa, y \in V_\kappa, z\in V_\kappa \land \phi^{V_\kappa}(y, z, \overline{w}))$, de sorte qu'on obtient $S\in V$ tel que $V\models \forall t, t\in S\iff \exists y \in x, \psi(y,t,\overline{w})$.

    En particulier, comme $V\models \psi(y,t,\overline{w}) \implies t\in V_\kappa$, on a $V\models S\subset V_\kappa$. De plus, $V\models |S|\leq |x|$. Or $V\models \forall e, e\in V_\kappa \implies |e| < \kappa$ (facile à montrer par récurrence, je pourrai donner des détails si besoin)

    En particulier, $V\models [S| < \kappa$. Bon, soit maintenant $rg$ l'application définie dans $V$ par $rg(q) = \min\{\alpha : q\in V_\alpha \}$. Remarque que $rg(q)$ est toujours successeur (d'ailleurs pour éviter cet écueil certains auteurs mettent un $\alpha +1$. Ici ça ne change rien). Alors(tout ça est dans $V$, mais flemme de continuer à écrire $V\models$ à chaque fois) $rg_{\mid S}$ est une application d'un ensemble de cardinal $<\kappa$ dans $\kappa$ (car $S\subset V_\kappa$). Par régularité de $\kappa$, elle est donc bornée, disons par $\lambda$. Donc $S\subset V_\lambda$, et donc $S\in V_{\lambda+1}$, $S\in V_\kappa$. On vérifie alors que $V_\kappa \models \forall t, t\in S\iff \exists y \in x, \phi(y,t, \overline{w})$.

    Finalement, $(V_\kappa, \in ) \models$ axiome de remplacement en $\phi$.

    Donc $(V_\kappa, \in) \models ZFC$.

    Faites moi savoir toutes les coquilles et éventuelles erreurs dans ce que j'ai écrit + @incohérence si tu veux encore des précisions, n'hésite pas à demander !
  • Juste un détail: y a-t-il un endroit, incohérence, où tu as demandé spécifiquement l'axiome "il existe un inaccessible". Parce que je lis ci-dessus maxtimax qui se plie en 4 avec CET axiome précis. Mais comme je l'ai dit, on ne le voit pas dans ta demande?

    Sinon, pour information, ZF est extrêmement redondant et c'est juste par respect de l'histoire qu'on garde la liste classique. Pour tes demandes, une version plus ramassée serait préférable: le sch remplacement => le sch de compréhension réduit de ZF et l'axiome des parties fait à peu près tout le reste. Par exemple $\{x;y\}$ s'obtient par remplacement à partir de $\{0;1\}$ et $1=\{0\} = P(0)$, etc.
    Aide les autres comme toi-même car ils sont toi, ils sont vraiment toi
  • @christophe : oui mais comme j'ai prouvé que $(V_\alpha, \in) \models ZC$ avant de faire l'hypothèse que $\alpha$ était en fait un inaccessible, j'ai préféré faire toute la liste (en perdant le remplacement on perd beaucoup de ces implications que tu mentionnes; et je n'ai jamais essayé de tester les redondances dans ZC)

    Sinon, j'ai annoncé que je ferais avec l'hypothèse "inaccessible" précédemment si ça peut te rassurer
  • Merci. J'adressais surtout ma question à incohérence toi j'ai bien vu que tu sais où tu vas (et même sans le voir je l'aurais deviné :-D ). Mais elle n'a toujours pas édite son premier post pour préciser. Cela dit l'outil édit ....
    Aide les autres comme toi-même car ils sont toi, ils sont vraiment toi
  • Merci pour la réponse détaillée Maxtimax ! Je ne l'ai que survolée pour le moment mais dès que j'ai des remarques ou questions je les poserai. :)
  • @Maxtimax: Merci pour ce message sur les cardinaux inaccessibles ! Est-ce que tu peux expliquer ce qu'est la relativisée de $\phi$ à $V_{\kappa}$ ? C'est la restriction ?
    Est-ce que $\forall \overline{w}$ remplace $\forall w_1, \dots,\forall w_n$ ?

    Est-ce que l'on a montré l'indépendance de l'axiome: "il existe un cardinal inaccessible" ?
  • @marco : Plus généralement, si $A$ est une formule à une variable libre représentant une classe transitive (i.e. $V\models A(x) \land y\in x \implies A(y)$), à paramètres dans $V$, la relativisée de $\phi$ à $A$, notée $\phi^A$ est définie par induction sur la complexité des formules:
    Si $\phi$ est atomique, $\phi^A = \phi$, si $\phi = \psi\land \chi$, $\phi^A = \psi^A \land \chi^A$, si $\phi = \neg \psi$, $\phi^A = \neg \psi^A$, et (c'est là qu'est l'intérêt) si $\phi = \exists x \psi$, alors $\phi^A = \exists x, A(x) \land \psi^A(x)$.

    Donc ici, comme $V_\alpha$ est définissable par une formule (notée aussi $V_\alpha$ par abus de notation) avec pour seul paramètre $\alpha$, on a une définition de formule relativisée à $V_\alpha$.

    Oui, j'utilise $\overline{w}$ pour me simplifier les notations : $\overline{w}$ représente un $n$-uplet $w_1,...,w_n$ (où $n$ est clair par le contexte), et $\forall \overline{w}$ abbrévie $\forall w_1,...,\forall w_n$.

    Alors dans ce fil j'ai presque montré le théorème suivant : "Si ZFC est cohérente alors ZFC ne démontre pas l'existence de cardinaux inaccessibles" (qui est un véritable théorème); par contre, si ZFC est cohérente, on ne peut pas démontrer "Si ZFC est cohérente alors ZFC ne démontre pas l'inexistence de cardinaux inaccessibles", sauf si l'existence de tels cardinaux est réfutable dans ZFC.

    En fait c'est ce que j'expliquais (très brièvement) dans mon post précédent: avec un tout petit peu de modifications sur la preuve que j'ai fournie, on obtient le théorème "Si $\kappa$ est inaccessible, $V_\kappa \models$ ZFC" , et alors on obtient "CI implique que ZFC est cohérente" (où CI est l'énoncé "il existe un cardinal inaccessible") en tant que théorème de ZFC, et donc ZFC + CI prouve "ZFC est cohérente".

    Ainsi, si on pouvait prouver "Si ZFC est cohérente, alors ZFC + CI l'est aussi", on aurait, comme théorème de ZFC + CI, "ZFC + CI est cohérente", ce qui impliquerait par le second théorème d'incomplétude de Gödel que ZFC + CI est incohérente.
  • @Maxtimax: merci pour les explications sur la relativisée et sur $\forall \overline{w}$.
    Au sujet de la deuxième partie de ton message, je dois encore y réfléchir car je ne suis pas très clair sur la question suivante (qui est un peu hors-sujet et paraîtra sans doute évidente): si on a un modèle d'une théorie axiomatique T dans ZFC, alors on peut démontrer l'énoncé "T est cohérente" dans ZFC.

    L'énoncé "T est cohérente" est-il un énoncé arithmétique que l'on plonge dans ZFC, ou est-ce autre chose ?

    Pour montrer le second théorème de Gödel (à savoir ici: si ZFC est cohérente, on ne peut pas montrer dans ZFC l'énoncé "ZFC est cohérente"), utilse-t-on nécessairement l'arithmétique ?
  • @marco : si ZFC démontre "$T$ a un modèle" alors par définition ZFC démontre "$T$ est consistante" et donc par le théorème de correction, ZFC démontre "$T$ est cohérente", oui.

    "$T$ est cohérente", si $T$ est récursivement axiomatisable est un énoncé qui ne concerne que $V_\omega$ donc on peut le qualifier d' "arithmétique", oui (notamment car $V_\omega$ est interprétable dans $\mathbb{N}$)

    Bah on peut le faire dans ZFC sans jamais parler d'arithmétique et on peut le faire dans l'arithmètique... il est compliqué de dire si on utilise nécessairement quelque chose dans une preuve
  • @Maxtimax: merci pour la réponse.
  • Une question bête sur les modèles : disons que l'ensemble $V$ que tu as construit est un arbre d'ensembles. J'ai du mal à voir comment on peut utiliser $V$ pour assigner vrai ou faux aux formules de ZFC.

    Est-ce que implicitement les nœuds de $V$ sont étiquetés par les formules de ZFC, ou bien est-ce qu'il faut utiliser une sorte de super oracle pour savoir s'il existe dans $V$ un nœud qui satisfait telle formule ?
  • @reuns : je ne l'ai pas construit $V$, je l'ai supposé donné (et encore heureux, sinon j'aurais prouvé que ZFC est incohérente :-D)
    Je vois pas ce que tu entends par "arbre d'ensembles" :-S

    Après pour assigner vrai ou faux aux formules de ZFC c'est par induction sur la complexité des formules. D'un point de vue informatique, j'imagine que tu supposes en quelque sorte que tu as, oui, une sorte de super oracle qui est capable de vérifier à chaque fois si telle formule est vraie; mais je ne saurais pas t'en dire plus parce que je ne comprends pas trop ta question
  • Au sujet de la question (1): si T a un modèle dans ZFC, alors ZFC démontre "T est cohérente". Je crois l'avoir démontré de la façon suivante.

    On démontre d'abord la proposition (2) suivante valable pour un énoncé P quelconque de T: si ZFC démontre "T démontre P" et si T a un modèle dans ZFC, alors ZFC démontre $\phi(P)$ où $\phi(P)$ est l'interprétation de P dans ZFC.
    On raisonne par l'absurde, en choisissant un énoncé P qui rende (2) faux, c'est-à-dire tel que ZFC démontre "T démontre P", mais tel que ZFC ne démontre pas $\phi(P)$. On choisit de plus P de sorte que la longueur de la démonstration de P dans T soit minimum. Alors il existe des énoncés P1 et P2 dans la preuve de P dans T à partir desquels on déduit P. De plus, la longueur de la démonstration de P1 et de P2 dans T est strictement plus petite que celle de P, donc on a $\phi(P_1)$ et $\phi(P_2)$, et donc on déduit $\phi(P)$, ce qui est une contradiction.
    De plus, pour tous les axiomes A de T, on a $\phi(A)$ par hypothèse car T a un modèle dans ZFC.
    Donc (2) est vrai.

    Pour démontrer (1), on choisit P=Faux et on prend la contraposée de (2). ZFC ne démontre pas $\phi(Faux)=Faux$, donc ZFC ne démontre pas "T démontre Faux", donc ZFC ne démontre pas "T est incohérente". Mais c'est différent de: ZFC démontre "T est cohérente", donc ça ne marche pas. Comment conclure ?
  • Attention, qu'appelles-tu "$T$ a un modèle dans ZFC " ? Moi j'ai simplement dit que si on peut prouver que $T$ a un modèle alors on peut prouver que $T$ est cohérente
  • maxtimax a écrit:
    par contre, si ZFC est cohérente, on ne peut pas démontrer "Si ZFC est cohérente alors ZFC ne démontre pas l'inexistence de cardinaux inaccessibles"

    Je n'ai pas tout lu, suis épuisé (soirées, tafs). Mais on sait jamais peut-être ton erreur de frappe dans la citation précédente a-t-elle déstabilisé marco??? Je ne sais pas. En la corrigeant, peut-être les choses iront-elles un peu mieux

    @marco: les inaccessibles sont bien ordonnés et sont tous des modèles de ZF, donc le plus petit vérifie [ZF+pas de CI].

    Cet argument est très souple, on peut le faire à propos de plein de choses, ça évite de passer par Godel: voici d'ailleurs un petit jeu-argument édifiant. Passons une décennie à faire tout plein d'efforts pour créer un adjectif (une relation avec une définition sans paramètre) $R$ dont on estime que $R(A)$ exprime
    <<dans les bons univers assez hauts et bien pleins, il devrait exister un inaccessible $E$ tel que $E\models A>>$

    Prenons l'énoncé

    $$B: = \exists E\in Inac: E\models [\forall X: (R(X)\to [\exists F\in Inac: F\models X])]$$

    Alors le plus petit inaccessible vérifiant $B$ ne peut pas penser que $R(B)$. Sinon, il n'existe pas du tout d'inaccessible vérifiant $B$. Aucune de ces conclusions n'est satisfaisante pour estimer qu'on a gagné à l'issue de la décennie. Autrement dit en simplifié:

    $<<A\mapsto [A$ n'a pas de modèle juste parce que l'univers est trop petit (sinon, il devrait en avoir un)$]>>$

    n'est pas définissable. (Or c'est intuitivement un absolu beaucoup plus figé que l'ensemble vérité d'un univers (qui dépend de l'univers).
    Aide les autres comme toi-même car ils sont toi, ils sont vraiment toi
  • @Maxtimax: par "T a un modèle dans ZFC", j'entends seulement que T admet un modèle. C'est vrai que ce n'est pas la peine de préciser "dans ZFC", un modèle étant toujours dans ZFC (à moins qu'il y ait des modèles dans ZC ou ZF). Oui, je vois bien que si T admet un modèle, et si ZFC est cohérente, alors T est cohérente. Mais, cette preuve n'est-elle pas métamathématique ? En effet, c'est différent de ZFC $\implies$ "T est cohérente", car dans cette dernière implication, "T est cohérente" est un énoncé de théorie des ensembles ou d'arithmétique.
    Je ne connais pas tellement bien la logique.
  • @Christophe : je ne vois pas l'erreur :-S peut-être mon emploi de "on" au lieu de ZFC ?

    @marco : "$T$ a un modèle dans ZFC" n'a pas beaucoup de sens... d'ailleurs il n'y a pas de "et si ZFC est cohérente": si $T$ a un modèle, $T$ est cohérente. C'est un théorème mathématique, au même titre que "Si $K$ est un corps, il admet une clôture algébrique" ou je ne sais quoi.
  • De mon téléphone : ce n'est pas vraiment une coquille c'est juste que tu voulais dire "existence" et non pas "inexistence" comme l'a attesté ce que tu as dit ensuite dans le post que je n'ai copié dans la citation pardon. De plus (mais c'est autre chose) il ya du coup une subtile inversion d'idée car comme ZF+ci est STRICTEMENT PLUS FORT en un certain sens en poussant on peut presque dire ZF+non ci est "obligatoire" au niveau ZF et que ci constitue un palier de risque donc est "presque réfutable". Bon mais je ne veux pas compliquer. (En plus simple ce n'est pas Gödel qui dit que ZF ne démontre pas ce que tu dis qu'il ne démontre pas c'est ci^+ lui même. Bon comme c'est une coquille probable)
    Aide les autres comme toi-même car ils sont toi, ils sont vraiment toi
  • @christophe : tu es sûr ? Je pense que c'est bien ce que je voulais dire, c'est pour ça que je parlais de Gödel (ta version "corrigée" se prouve automatiquement, sans Gödel). J'ai bien écrit "on ne peut pas démontrer", alors qu'on peut très bien démontrer "Si ZFC est cohérente, ZFC ne démontre pas l'existence d'inaccessibles"
  • @max: non, je ne suis sûr de rien, d'autant que je n'ai pas lu le fil. C'est juste que j'ai saisi au bon la phrase:
    <<si ZF est consistant alors ZF ne démontre pas [large]l'in[/large]existence de cardinaux inaccessibles>>

    et en lisant en diagonale la suite, j'ai vu que tu évoquais le fait que par Godel, ZF (si consistant) ne démontre pas [size=x-large]l'e[/size]xistence de cardinaux inaccessibles et que tu développais un certain nombre d'explications à ce propos adressés à marco et incohérence.

    Pour te répondre correctement, il faudrait que je prenne 10 bonnes minutes à tout analyser à la fois de leurs questions et de ta réponse. L'important est que toi, tu aies pu préciser une deuxième fois, que tu n'estimes pas avoir fait de coquille de sorte que ça permet à tout le monde de ne pas se retrouver devant une (EVENTUELLE) coquille subliminale.

    Quant à moi, je précise mon propos de mon téléphone: 38 prouve la consistance de 25 ou de 37, mais pas de 38 par Godel. Par contre 39, 40, 41, etc sont dans les vapeurs du ciel :-D, en ce sens qu'on n'affirme pas la consistance de 39 à l'étage 38 (ni la consistance de 540, 100, 158, etc) par Godel ou quelqu' argument scientifique que ce soit. Ce sont des "actes de foi" (non religieux). Gödel dit juste qu'au niveau n, on n'a pas la possibilité de prouver les consistance des niveaux p dès lors que $p\geq n$. Des "actes de foi" platoniciens conduisent un certain nombre de scientifiques, qui jusqu'alors parlaient au niveau $n$ à affirmer la consistance du niveau $n+1$ (et même $n$).

    Quand je dis Gödel, c'est dans un sens très général. La théorie des ensembles a trivialisé le godélisme et évacué le besoin de coder ou d'utiliser un argument diagonal, comme je l'ai dit plus haut, puisque le plus petit gros machin bleu est un modèle de ZF qui ne contient de pas de gros machin bleu, ce qui établit la consistance de "ZF + il n'existe pas de gros machins bleu" en une ligne évidente. Ca a bien sûr ses limites (mais elles sont loin), puisqu'il faut changer (très légèrement) de paradigme et se restreindre aux modèles bien fondés. Par exemple, il existe une suite $T$ de théories récursiveS telle que pour tout $n\in \N: T_n\vdash cons(T_{n+1})$, mais évidemment tout le monde s'accorde pour dire qu'elles sont artificielles.
    Aide les autres comme toi-même car ils sont toi, ils sont vraiment toi
  • @max, j'espère que tu vas bien (ça fait longtemps que tu n'as pas posté). Bon, je n'ai toujours pas eu le temps d'examiner en entier le fil, par contre, je recopie, pour la commenter ta citation tout entière, ie tout le paragraphe:
    maxtimax a écrit:
    Alors dans ce fil j'ai presque montré le théorème suivant : "Si ZFC est cohérente alors ZFC ne démontre pas l'existence de cardinaux inaccessibles" (qui est un véritable théorème); par contre, si ZFC est cohérente, on ne peut pas démontrer "Si ZFC est cohérente alors ZFC ne démontre pas l'inexistence de cardinaux inaccessibles", sauf si l'existence de tels cardinaux est réfutable dans ZFC.

    La première partie en noir est claire et comme déjà dit, le premier inaccessible est un modèle de [ZFC + no CI]

    Celle que j'ai mise en bleu est à tout le moins bizarre en ce qu'elle commence par "si ZFC est cohérente". En sémantisant, tu écris que si ZFC est cohérente alors il existe un modèle de ZFC qui croit à l'énoncé "ZFC est cohérente et ZFC prouve qu'il n'existe pas de CI" et tu ajoutes un "sauf si ZFC prouve qu'il n'y a pas de CI"

    Ce que tu affirmes n'est pas prouvable et de plus, quand vrai, c'est "vrai" même si ZFC ne prouve pas l'inexistence de CI de toute façon, le "sauf si"*** est déstabilisant me semble-t-il. De plus, si ZFC n'est pas cohérente, évidemment, ZFC prouve ce qu'on veut.

    En gros, pour que ce soit moins déstabilisant, tu aurais peut-être dû commencer par "si ZFC+cons(ZFC)" est cohérente alors " et ne pas mettre un "sauf si" à la fin (sauf erreur, je n'ai peut-être pas pris assez de temps).

    Je crois que je comprends le message que tu veux faire passer: c'est que ZFC est strictement moins forte que cons(ZFC) qui elle-même est strictement moins forte que ZFC+CI, qui elle-même est strictement moins forte que cons(ZFC+CI), etc. C'est un message tout à fait correct, là n'est pas la question.

    *** qui peut être interprété comme un "ou"?
    Aide les autres comme toi-même car ils sont toi, ils sont vraiment toi
  • Ta conclusion est correcte, c'est le message que je voulais faire passer; mais j'avoue ne pas vraiment voir en quoi ce que j'ai écrit n'est pas prouvable. Je te fais une esquisse de preuve :
    On suppose ZFC cohérente, et on suppose que ce que tu cites est démontrable. On suppose de plus que ZFC ne réfute pas CI (c'est mon "sauf si"). Alors ZFC + CI démontre "Con(ZFC) implique con(ZFC + CI)" et "con(ZFC)" donc ZFC + CI démontre con(ZFC + CI); donc par Gödel ZFC + CI est incohérente. Par hypothèse ("ZFC ne réfute pas CI"), c'est absurde. Donc la partie entre-guillemets n'est pas démontrable.

    En écrivant ça je me rends compte qu'avec la fatigue il est très probable que je m'embrouille entre les "niveaux de ZFC" dont je parle et donc tu as potentiellement raison; mais il faudrait regarder ça proprement. D'ailleurs si ma "preuve" tient debout, l'hypothèse "ZFC est cohérente" est redondante puisque j'ai supposé que ZFC ne réfutait pas CI donc que ZFC était cohérente.

    Je m'essaie à la refaire proprement : je suppose $ZFC \vdash Con(ZFC) \implies Con(ZFC + CI)$, et $ZFC \nvdash \neg CI$. Alors $ZFC + CI \vdash Con(ZFC + CI)$ car $ZFC + CI\vdash Con(ZFC)$. Donc par Gödel, $ZFC + CI \vdash \bot$, donc $ZFC \vdash \neg CI$, absurde. Donc, sauf si $ZFC \vdash \neg CI$, $ZFC \nvdash con(ZFC) \implies con(ZFC + CI)$.

    Non ?
  • De mon téléphone : moi aussi je m'embrouille (comme tout le monde je pense d'ailleurs sur ces sujets là; c'est pointilleux). Mais quand je lis que tu supposes que zfc prouve (cons(zfc)=> cons(zfc+ci)) j'ai un doute sur tes parenthèses ça revient à supposer que zfc prouve non cons(zfc). Donc effectivement si je n'ai pas vu que tu fais cette hypothèse pardon car elle est forte. Je relirai d'un PC
    Aide les autres comme toi-même car ils sont toi, ils sont vraiment toi
  • Je vins de relire attentivement même si de mon téléphone. Donc oui tu as raison ton affirmation bleue est prouvable dès lors qui'on lit "sauf si" comme synonyme de "ou" inclusif. Mais elle est tellement un enchainement de négations de negation s etc qu'elle est "trop faible" par rapport à la réalité sous jacente qui est que ZFC ne peut prouver ( consZFC) => cons(ZFC+CI) que si ZFC prouve non (consZFC). Et tout ceci se passe tres en dessous des CI. Je pense que tu faisais une réponse contextuee. Par contre il ne faut pas voir le "sauf si" comme en français qui recherche une minimalité: il se peut très bien que ZFC prouve NOCI mais que ses consistances itérées soient toutes vraies par exemple. Il se peut aussi que ZFC prouve non cons(ZFC+CI) sans prouver nonCI. Etc,etc.

    Si on veut résumer ce sue tu as dit dans un slogan: "T ne prouve pas que cons(A)=>cons(B)" quand A<B sauf quand T+A déconne à un ou deux étages près donc n'est pas une théorie crédible.
    Aide les autres comme toi-même car ils sont toi, ils sont vraiment toi
  • Cette hypothèse est peut-être trop forte mais elle représente bien ce que je voulais dire il me semble. Contrairement par exemple à $ZF \vdash con(ZF)\implies con(ZFC)$ etc.; on voit que CI est "d'un autre niveau", et ce niveau c'est précisément une consistency strength strictement supérieure. Mais je me doute qu'il y a des résultats plus forts. Je suis d'accord que mon "sauf si" était peut-être mal formulé. Maintenant qu'on a dit tout ça, ce que je voulais dire et ce qui est vrai est clair ! Merci pour tes clarifications (le fait que tu n'aies pas tout de suite vu ce que je voulais dire montre qu'il était important de clarifier !)
  • Je ne suis pas une référence en soin, ni même en "théorie godélienne classique**" spécialement, ça aurait été à moi de faire plus attention et de ne commenter un tel item qu'après avoir parsé en détails la chose. D'ailleurs, pour ma décharge, je crois me rappeler que quand j'ai écrit "n'est pas prouvable", j'avais en fait éliminé la partie "sauf si" (mais n'ai pas dû le préciser). De même à mon dernier post, j'ai oublié le $<<T\leq A>>$ dans $<<T \leq A<B>>$.

    Par contre, où je ne suis pas tout à fait d'accord avec toi (ou du moins où je ne comprends pas ta motivation), c'est quand tu écris que tu "préfères" dire "si $ZFC$ ne prouve pas nonCI alors ...", plutôt que "si $ZFC$ ne prouve pas $non(cons(ZFC))$ alors ...". :-S . Bon remarque, ce n'est pas vraiment non plus ce que tu écris à vrai dire. J'ajoute quelques infos:

    1) [ZFC + il existe un modèle de ZFC dans lequel il n'y a pas d'entiers infinis] est énooooooooooooooooooooormééééééééééément plus fort que [ZFC + il existe un modèle de ZFC]

    2) [ZFC + il existe un modèle bien fondé de ZFC] est énooooooooooooooooooooormééééééééééément plus fort que [ZFC + il existe un modèle de ZFC dans lequel il n'y a pas d'entiers infinis]

    3) [ZFC + il existe un CI] est énooooooooooooooooooooormééééééééééément plus fort que [ZFC + il existe un modèle bien fondé de ZFC]

    Bref, [ZFC + cons(ZFC+cons(ZFC+cons(ZFC))...)] est une petite joueuse à côté :-D

    ** j'ai développé la mienne, personnelle.
    Aide les autres comme toi-même car ils sont toi, ils sont vraiment toi
  • Bah je préfère dire ça au sens où les inaccessibles sont soit réfutables, soit "inutilisables sans scrupule"
  • :-D Pour faire un deux bons mots, je dirais que:

    1) ils font pâle figure à côté des grosses bébêtes qu'on a aujourd'hui (par exemple, les $n-huge$

    2) je reviens à "sauf si", ça me fait penser à la blague hyperconnue $<<$ tu auras une baffe.. sauf si tu manges ta soupe. Bébé mange sa soupe et ... prend une baffe $>> $ :-D
    Aide les autres comme toi-même car ils sont toi, ils sont vraiment toi
Connectez-vous ou Inscrivez-vous pour répondre.
Success message!